Difference between revisions of "2010 AIME II Problems/Problem 7"

m (Solution 2 (casework))
Line 34: Line 34:
 
Case 3: <math>2w-4</math> is real. Going through the same steps, we get <math>y=0</math>, so <math>w=x</math>. Now <math>w+3i</math> and <math>w+6i</math> are complex conjugates, but <math>w=x</math>, which means that <math>\overline{x+3i}=x+6i</math>, so <math>x-3i=x+6i</math>, which has no solutions.
 
Case 3: <math>2w-4</math> is real. Going through the same steps, we get <math>y=0</math>, so <math>w=x</math>. Now <math>w+3i</math> and <math>w+6i</math> are complex conjugates, but <math>w=x</math>, which means that <math>\overline{x+3i}=x+6i</math>, so <math>x-3i=x+6i</math>, which has no solutions.
  
Thus case 1 is the only one that works, so <math>w=4-3i</math> and our polynomial is <math>(x-(4))(x-(4+6i))(x-(4-6i))</math>. Note that instead of expanding this, we can save time by realizing that the answer format is <math>|a+b+c|</math>, so we can plug in <math>x=1</math> to our polynomial to get the sum of coefficients, which will give us <math>a+b+c+1</math>. Plugging in <math>x=-1</math> into our polynomial, we get <math>(-3)(-3-6i)(-3+6i)</math> which evaluates to <math>-135</math>. Since this is <math>a+b+c+1</math>, we subtract 1 from this to get <math>a+b+c=-136</math>, so <math>|a+b+c|=\boxed{136}</math>.
+
Thus case 1 is the only one that works, so <math>w=4-3i</math> and our polynomial is <math>(z-(4))(z-(4+6i))(z-(4-6i))</math>. Note that instead of expanding this, we can save time by realizing that the answer format is <math>|a+b+c|</math>, so we can plug in <math>z=1</math> to our polynomial to get the sum of coefficients, which will give us <math>a+b+c+1</math>. Plugging in <math>z=1</math> into our polynomial, we get <math>(-3)(-3-6i)(-3+6i)</math> which evaluates to <math>-135</math>. Since this is <math>a+b+c+1</math>, we subtract 1 from this to get <math>a+b+c=-136</math>, so <math>|a+b+c|=\boxed{136}</math>.
  
 
~chrisdiamond10
 
~chrisdiamond10

Revision as of 01:15, 12 February 2021

Problem 7

Let $P(z)=z^3+az^2+bz+c$, where a, b, and c are real. There exists a complex number $w$ such that the three roots of $P(z)$ are $w+3i$, $w+9i$, and $2w-4$, where $i^2=-1$. Find $|a+b+c|$.

Solution (vieta's)

Set $w=x+yi$, so $x_1 = x+(y+3)i$, $x_2 = x+(y+9)i$, $x_3 = 2x-4+2yi$.

Since $a,b,c\in{R}$, the imaginary part of $a,b,c$ must be $0$.

Start with a, since it's the easiest one to do: $y+3+y+9+2y=0, y=-3$,

and therefore: $x_1 = x$, $x_2 = x+6i$, $x_3 = 2x-4-6i$.

Now, do the part where the imaginary part of c is 0 since it's the second easiest one to do: $x(x+6i)(2x-4-6i)$. The imaginary part is $6x^2-24x$, which is 0, and therefore $x=4$, since $x=0$ doesn't work.

So now, $x_1 = 4, x_2 = 4+6i, x_3 = 4-6i$,

and therefore: $a=-12, b=84, c=-208$. Finally, we have $|a+b+c|=|-12+84-208|=\boxed{136}$.

Solution 1b

Same as solution 1 except that when you get to $x_1 = x$, $x_2 = x+6i$, $x_3 = 2x-4-6i$, you don't need to find the imaginary part of $c$. We know that $x_1$ is a real number, which means that $x_2$ and $x_3$ are complex conjugates. Therefore, $x=2x-4$.

Solution 2 (casework)

Note that at least one of $w+3i$, $w+9i$, and $2w-4$ is real by complex conjugate roots. We now separate into casework based on which one.

Let $w=x+yi$, where $x$ and $y$ are reals.

Case 1: $w+3i$ is real. This implies that $x+yi+3i$ is real, so by setting the imaginary part equal to zero we get $y=-3$, so $w=x-3i$. Now note that since $w+3i$ is real, $w+9i$ and $2w-4$ are complex conjugates. Thus $\overline{w+9i}=2w-4$, so $\overline{x+6i}=2(x-3i)-4$, implying that $x=4$, so $w=4-3i$.

Case 2: $w+9i$ is real. This means that $x+yi+9i$ is real, so again setting imaginary part to zero we get $y=-9$, so $w=x-9i$. Now by the same logic as above $w+3i$ and $2w-4$ are complex conjugates. Thus $\overline{w+3i}=2w-4$, so $\overline{x-6i}=2(x-9i)-4$, so $x+6i=2x-4-18i$, which has no solution as $x$ is real.

Case 3: $2w-4$ is real. Going through the same steps, we get $y=0$, so $w=x$. Now $w+3i$ and $w+6i$ are complex conjugates, but $w=x$, which means that $\overline{x+3i}=x+6i$, so $x-3i=x+6i$, which has no solutions.

Thus case 1 is the only one that works, so $w=4-3i$ and our polynomial is $(z-(4))(z-(4+6i))(z-(4-6i))$. Note that instead of expanding this, we can save time by realizing that the answer format is $|a+b+c|$, so we can plug in $z=1$ to our polynomial to get the sum of coefficients, which will give us $a+b+c+1$. Plugging in $z=1$ into our polynomial, we get $(-3)(-3-6i)(-3+6i)$ which evaluates to $-135$. Since this is $a+b+c+1$, we subtract 1 from this to get $a+b+c=-136$, so $|a+b+c|=\boxed{136}$.

~chrisdiamond10

See also

2010 AIME II (ProblemsAnswer KeyResources)
Preceded by
Problem 6
Followed by
Problem 8
1 2 3 4 5 6 7 8 9 10 11 12 13 14 15
All AIME Problems and Solutions

The problems on this page are copyrighted by the Mathematical Association of America's American Mathematics Competitions. AMC logo.png